Problema - Demostración de la conservación de la energía electromagnética

Quiero resolver el siguiente problema de mi libro de electrodinámica clásica:

Considere un cuerpo de masa pontual, metro , con cargo q , que se mueve con una velocidad uniforme v en una región donde existe un campo electromagnético. Demostrar que la energía total de la partícula y el campo electromagnético:

W = 1 2 metro v 2 + 1 2 ( ε mi 2 + B 2 m ) d V

es una constante de movimiento.

Y ahora, les mostraré algunas secciones de la resolución del libro que no entendí:

Comenzamos derivando la expresión de energía en orden al tiempo, asumiendo que V es fijo y las características de la media son independientes del tiempo:

d W d t = metro v d v d t + V ( ε mi mi t + B m B t ) d V =
metro ( v . d v d t ) + V [ ( mi . D d t ) + ( H . B d t ) ] d V

. . .

j = ρ v y q = V ρ d V entonces:

V ( mi . j ) d V = ( mi . v ) q

. . .

Tengo las siguientes dudas:

  1. ¿Podemos escribir por ejemplo d mi d t = mi t ? El campo electromagnético también depende del vector de posición. r . Entonces de la regla de la cadena tenemos: d mi d t = mi r v + mi t
  2. Sé que la enunciación dice que v es uniforme Pero si hicieron todo el trabajo que mostré entonces creo que asumieron que d v d t no es necesariamente 0 . ¿Es correcto escribir v d v d t = ( v . d v d t ) ? por ejemplo si v = ( pecado ( t ) , porque ( t ) , 0 ) entonces d v d t = ( porque ( t ) , pecado ( t ) , 0 ) . Entonces, v d v d t = 1 y por otra parte ( v . d v d t ) = 0 Que es diferente.
  3. ¿Es correcto escribir V ( mi . j ) d V = ( mi . v ) q ? Esto significa que mi . v es una constante que no puede ser del todo cierta.

[PD: Solo estoy publicando esta pregunta con todos esos detalles, porque en el examen si viera esta pregunta comenzaría a resolverla de la manera más general, y no asumiendo todas esas condiciones que tiene la resolución. Pero la verdad es que si no asumiera esas condiciones fracasaría. Entonces, solo necesito entender esos detalles.]

Respuestas (1)

  1. ¿Podemos escribir por ejemplo d mi d t = mi t ?

Intenta escribir una expresión para W ( t 2 ) y W ( t 1 ) y restándolos y luego dividiendo el resultado por t 2 t 1 . Si luego empuja las partes con los campos dentro de una integral común, verá algo que se parece a / t ?

Nunca dejes que los símbolos te confundan, son notación para expresar ideas. El resultado: d mi d t = mi X X ^ v + mi y y ^ v + mi z z ^ v + mi t está calculando la tasa de cambio de tiempo de GRAMO ( t ) = mi ( r ( t ) , t ) . Pero estás evaluando el campo en cada punto, en todas partes del espacio, para hacer una integral sobre todo el espacio. Así que solo calcule esa integral para dos tiempos diferentes, reste y escale, luego tome el límite.

¿Es correcto escribir v d v d t = ( v . d v d t ) ?

El primero es solo un error. Computar d d t ( v v ) usted obtiene ( v d d t v ) + ( d d t v ) v por la regla del producto que es igual 2 ( v d d t v ) de la simetría del producto escalar. Así que el punto siempre debería haber estado ahí. Si no conoces esta regla, deja que a = X ^ v , b = y ^ v , y C = z ^ v entonces

d d t ( v v ) = d d t ( a 2 + b 2 + C 2 ) = 2 ( a d a d t + b d b d t + C d C d t ) .

Si su referencia va a omitir puntos y luego agregarlos nuevamente como si fuera un paso, entonces asegúrese de saber lo que haría y aprenda a leer el texto como una especie de copia incorrecta del trabajo correcto, que usted debe ser capaz de hacer usted mismo.

  1. ¿Es correcto escribir V ( mi . j ) d V = ( mi . v ) q ? Esto significa que mi . v es una constante que no puede ser del todo cierta.

Asegúrese de que puede articular la diferencia entre constante (en el tiempo) y uniforme (en el espacio) y la velocidad no debe ser ninguna de las dos. Si j = q v d ( r ( t ) ) entonces V ( mi j ) d V = mi ( r ( t ) , t ) ( q v ( t ) ) evaluando el delta de Dirac.

¿Podrías explicar mejor el 3er punto?
@ÉlioPereira ¿Ya estudiaste las distribuciones delta de Dirac?
No, no lo hice. Estoy empezando Física Matemática este semestre.
@ÉlioPereira Entonces no sé cómo estabas leyendo j ( X , t ) = ρ ( X , t ) v ( t ) dado que el primero es un campo y el segundo es un solo vector, necesita una corriente que sea cero en todas partes excepto donde está la carga y da ρ ( X , t ) d v = q , si imagina un límite de gaussianas con desviaciones estándar cada vez más pequeñas, prácticamente está allí. Si profundiza en la física matemática, es posible que desee aprender sobre derivadas débiles y luego distribuciones como funcionales acotados en funciones de prueba. Luego aparece allí también la distribución de Dirac.